timohuu 发表于 2012-9-15 22:00:24

第一篇ARGUMENT,求拍,留下链接必回拍。。。。。


写的是ARGUMENT55,求拍,留下链接必回拍。。。。


55.The following appeared in an editorial in a business magazine.
"Although the sales of Whirlwind video games have declined over the past two years, a recent survey of video-game players suggests that this sales trend is about to be reversed. The survey asked video-game players what features they thought were most important in a video game. According to the survey, players prefer games that provide lifelike graphics, which require the most up-to-date computers. Whirlwind has just introduced several such games with an extensive advertising campaign directed at people ten to twenty-five years old, the age-group most likely to play video games. It follows, then, that the sales of Whirlwind video games are likely to increase dramatically in the next few months."
Write a response in which you examine the stated and/or unstated assumptions of the argument. Be sure to explain how the argument depends on these assumptions and what the implications are for the argument if the assumptions prove unwarranted.

The arguer's statement of Whirlwind's video games sales tend to increase greatly in the next few months, which seems to be perfectly logical and reasonable at first glance. However, after carefully reevaluating assumptions the author mentiones, we may consequently draw a conclusion that this assertion is totally invalid. To make the argument more convincing, additional is needed.

To begin with, citing a survey of video-players, the arguer presents that video sales is surely to be reversed. It's not clear, however, the scope and effectiveness of this survey. Perhaps very few video-game players have been taken into account in this survey, or perhaps those interviwees haven't told the truth. Also the investigators may be driven by some benefits and of course show baias to the survey. For that matter, the unrepresentativeness, bootless and unreliable investigation cannot bolster the author's argument.

In addition, the author implies people ten to twenty-five years old will purchase Whirlwind's new kind of games because of this age-group's preference to video games as well as Whirlwind's extensive advertising campaign. We do not know the exact price of this video games. There exist an accasion that the price of video games is relatively high .Commonsense tells us  people age at ten to twenty-five are mostly school students and they seems to have not enough money to buy this games. Aside from this, if Whirlwind's advertisement is just based on magazines that people do not prefer to. On this condition, those younth seems to fail to be influenced by this advertising campaign and would not buy the video games. To strenthen his/her viewpoint, it's essential that the arguer needs to cites more telling evidence.

Furthermore, building upon the assumption tendered above, the author concludes the sales of Whirlwind video games will truely incrsease. Yet it's unknown that the local economic condition. Take an economic crisis as an excample, the rise of globalization, both economy and cultural that has swept thoughout the world. Duing to globalization trend,the economic crisis broke out in other countries will really has a strong impact on local economic. What's more, if Whirlwind's competitor has already invented video games that tremendous cheaper and appealling to younthters. It's possible that the the younth will not buy Whirlwind's video games. Regardless what the arguer has presented, this statement fails to show the increasing sales of video games.

In sum, that the arguer comes to the conclusion is misleading and effectiveness. To make his assertion more persutable, other extra evidences are of considerable importance.
页: [1]
查看完整版本: 第一篇ARGUMENT,求拍,留下链接必回拍。。。。。